intTypePromotion=1
zunia.vn Tuyển sinh 2024 dành cho Gen-Z zunia.vn zunia.vn
ADSENSE

Kiểm tra cấu trúc ( G,* )

Chia sẻ: Lan Lan | Ngày: | Loại File: PDF | Số trang:20

96
lượt xem
10
download
 
  Download Vui lòng tải xuống để xem tài liệu đầy đủ

Bài 1: kiểm tra cấu trúc ( G,* ) có là nửa nhóm,vị nhóm hay nhóm không, và xét tính giao hoán của chúng.Trường hợp ( G,*) là nhóm, hãy mô tả các phần tử có cấp hữu hạn của nhóm này. a) G=Q\{-6},x*y = 90xy+540x+540y+3234 = 90(x+6)(y+6)-6. • Tính kết hợp: (x*y) *z = (90(x+6)(y+6)-6) *z = 90(90(x+6)(y+6)-6+6)(z+6)-6 (1) = 902(x+6)(y+6)(z+6)-6 x*(y*z) = x*(90(y+6)(z+6)-6) = 90(x+6)( 90(y+6)(z+6)-6+6)-6 = 902(x+6)(y+6)(z+6)-6 (2) từ (1) và (2) suy ra ( G,* ) có tính kết hợp ( G,* ) là nửa nhóm. • Phần tử trung hòa của trên G: x*e=e*x=x ...

Chủ đề:
Lưu

Nội dung Text: Kiểm tra cấu trúc ( G,* )

  1. Nhóm 1 Bài 1: kiểm tra cấu trúc ( G,* ) có là nửa nhóm,vị nhóm hay nhóm không, và xét tính giao hoán của chúng.Trường hợp ( G,*) là nhóm, hãy mô tả các phần tử có cấp hữu hạn của nhóm này. a) G=Q\{-6},x*y = 90xy+540x+540y+3234 = 90(x+6)(y+6)-6. • Tính kết hợp: (x*y) *z = (90(x+6)(y+6)-6) *z = 90(90(x+6)(y+6)-6+6)(z+6)-6 = 902(x+6)(y+6)(z+6)-6 (1) x*(y*z) = x*(90(y+6)(z+6)-6) = 90(x+6)( 90(y+6)(z+6)-6+6)-6 = 902(x+6)(y+6)(z+6)-6 (2) từ (1) và (2) suy ra ( G,* ) có tính kết hợp ( G,* ) là nửa nhóm. • Phần tử trung hòa của trên G: x*e=e*x=x 90(x+6)(e+6)-6 = 90(e+6)(x+6)-6 = x 90(x+6)(e+6) = 90(e+6)(x+6) = x+6 1 e = -6 ∈ Q\{-6 } 90 1 ( G,* ) là nửa nhóm có phần tử trung trung hòa e = -6 ∈ Q\{-6} 90 suy ra ( G,* ) là vị nhóm. • Phần tử x-1 của x: x*x-1 = x-1*x = e hay 1 90(x+6)(x-1+6)-6 = 90(x-1+6)(x+6)-6 = -6 90 1 x-1 = − 6 ∈ Q\{-6 } 90 ( x + 6) 2 suy ra ( G,* ) là nhóm • Do tính giao hoán của phép nhân trong Q ta có x*y = y*x = 90(x+6)(y+6)-6 suy ra ( G,* ) là nhóm giao hoán. • Cấp hữu hạn của phần tử x trong (G,*) : Với k ∈ N* bằng quy nạp ta chứng minh được xk = 90k-1 (x+6)k – 6 1 90k-1 (x+6)k – 6 = -6 90 [90(x+6)]k = 1 90(x+6) = 1 hoặc 90(x+6) = -1 vaø k =2 (do k ∈ N*,xk = e) 1
  2. 1 1 x= - 6 (=e) hoaëc x = - - 6 vaø caáp cuûa x laø 2 90 90 Câu b cm tương tự. c) G = R, x*y = (xn+yn)1/n, trong đó n là số nguyên dương lẻ cho trước. • Tính kết hợp: (x*y)*z = (xn+yn)1/n*z = (xn+yn+zn)1/n x*(y*z) = x*(yn+zn)1/n = (xn+yn+zn)1/n suy ra (G,*) có tính kết hợp nên (G,*) là nửa nhóm. • Phần tử trung hòa của trên G: x*e=e*x=x (x +e ) = (en+xn)1/n = x n n 1/n xn+en = xn en =0 e = 0 ∈ R (do n là số nguyên dương cho trước) suy ra ( G,* ) là vị nhóm. • Phần tử x-1 của x: x*x-1 = x-1*x = e hay (xn+x-n)1/n = e x-n = e- xn = - xn x-1 = -x suy ra ( G,* ) là nhóm • Tính giao hoán : x*y = (xn+yn)1/n = (yn+xn)1/n = y*x vậy (G,*) là nhóm giao hoán (nhóm abel) +)Cấp hữu hạn của phần tử x trong (G,*) : Với k ∈ N* bằng quy nạp ta chứng minh được xk = 2(k-1)/nx = 0 Suy ra x = 0 vậy (G,*) không có phần tử có cấp hữu hạn. Các câu d,e cm tương tự. f) G = R x R*,(x,y) * (z,t) = (x+yz,yt). • Tính kết hợp: ((x,y) * (z,t)) * (p,q) = (x+yz,yt) * (p,q) = (x+yz+ytp,ytq) (1) (x,y) * ((z,t) * (p,q)) = (x,y) * (z+tp,tq) = (x+y(z+tp),ytq) = (x+yz+ytp,ytq) (2) 2
  3. từ (1) và (2) suy ra ( G,* ) có tính kết hợp ( G,* ) là nửa nhóm. • Phần tử trung hòa của trên G: t*e=e*t=t với t = (x,y), e = (x0,y0) (x,y) * (x0,y0) = (x0,y0) * (x,y) = (x,y) (x+yx0,yy0) = (x0+y0x,y0y) = (x,y) x+y0x = x0+y0x = x và yy0 = y0y = y chọn y # 0 e = (x,1) suy ra phần tử e không duy nhất Vậy (G,*) không có phần tử trung hòa nên không là vị nhóm. +) Do (G,*) không là vị nhóm nên không có phần tử khả nghịch • Tính giao hoán : (x,y) * (z,t) = (x+yz,yt) (z,t) * (x,y) = (z+tx,ty) (x,y) * (z,t) # (z,t) * (x,y), • (G,*) không có tính giao hoán. • Do (G,*) không có phần tử trung hòa nên không tồn tại phần tử có cấp hữu hạn. g) G = R x R*,(x,y) * (z,t) = (xz-yt,xt+yz). • Tính kết hợp: ((x,y) * (z,t)) * (p,q) = (xz-yt,xt+yz) * (p,q) = ((xz-yt)p- (xt+yz)q, (xz-yt)q+(xt+yz)p) = (xzp-ytp-xtq-yzq,xzq-ytq+xtp+yzp) (1) (x,y) * ((z,t)* (p,q)) = (x,y) * (zp-tq,zq+tp) = (x(zp-tq)-y(zq+tp),x(zq+tp)+y(zp-tq)) = (xzp-ytp-xtq-yzq,xzq-ytq+xtp+yzp) (2) từ (1) và (2) ta có ((x,y) * (z,t)) * (p,q) = (x,y) * ((z,t)* (p,q)) (G,+) có tính kết hợp đối với phép * nên (G,*) là nửa nhóm. • Phần tử trung hòa của trên G: t*e=e*t=t với t = (x,y) và e = (x0,y0) (xx0-yy0,xy0+yx0) = (x0x-y0y,x0y + y0x) = (x,y) x(x0-1) - yy0 = 0 và y(x0-1) + xy0 =0 với mọi (x,y) ∈ R x R* (x0,y0) = (1,0) không thuộc vào R x R* suy ra ( G,* ) không là vị nhóm. • Do (G,*) không là vị nhóm nên không có phần tử khả nghịch • Tính giao hoán : với t=(x,y), t’ = (x’, y’) ta có t*t’ = (xx’-yy’,xy’+yx’) t’*t = (x’x-y’y,x’y + y’x) suy ra t*t’= t’*t nên (G,*) là nửa nhóm giao hoán. Câu h cm tương tự. Baøi 1.5: Cho G laø moät nhoùm trong ñoù coù duy nhaát moät phaàn töû a coù caáp 2. Chöùng minh raèng vôùi moïi x ∈ G thì ax = xa Giaûi: • Nhaéc laïi a caáp 2 nghóa laø a 2 = e vaø a ≠ e 3
  4. • Ta coù : (x −1 ax) 2 = x −1 ax x −1 ax = x −1a 2 x = x −1ex = x −1 x = e Hôn nöõa neáu x −1ax = e thì ax=xe=ex suy ra a=e (maâu thuaãn) Vaäy (x −1 ax) 2 =e vaø x −1ax ≠ e ñieàu naøy chöùng toû x −1ax coù caáp 2, maø a laø phaàn töû coù caáp 2 duy nhaát neân baét buoäc x −1ax =a suy ra xa=ax (ñpcm) Baøi 1.9. Chöùng minh raèng neáu (G,.) laø moät nhoùm giao hoaùn coù ñuùng n phaàn töû khaùc nhau laø x1 , x2 ,..., xn thì ( x1 x2 ...xn ) 2 = e . Giaûi: • Ñöông nhieân x1−1 , x2 −1 ,..., xn −1 ∈ G . Ta coù: e = ( x1 x1−1 )( x2 x2 −1 )...( xn xn −1 ) = ( x1 x2 ...xn )( x1−1 x2 −1...xn −1 ) ( do (G,.) laø 1 nhoùm giao hoaùn) • Ta caàn chöùng minh x1−1 x2 −1...xn −1 = x1 x2 ...xn nöõa laø xong! Giaû söû toàn taïi k ≠ l sao cho : x k −1 = xl −1 ( 0 < k < l ≤ n) Ta coù : x k . x k −1 = xl . xl −1 =e. Suy ra : x k = x l ( luaät giaûn öôùc) maâu thuaãn vôùi giaû thieát! Do ñoù : : x k −1 ≠ xl −1 vôùi k ≠ l. Hay {x1-1 ,x 2 -1 ,...,x n -1} laø 1 boä goàm n phaàn töû khaùc nhau trong G, maø G coù n phaàn töû laø x1 , x2 ,..., xn neân baét buoäc {x 1 -1 ,x 2 -1 ,...,x n -1} = {x1 ,x 2 ,...,x n } . Vaäy x1−1 x2 −1...xn −1 = x1 x2 ...xn Baøi toaùn giaûi quyeát xong. Baøi 1.13: Cho ví duï chöùng toû luyõ thöøa cuûa moät chu trình ko nhaát thieát phaûi laø chu trình. Giaûi : ⎛1234 ⎞ ⎛1234 ⎞ Ta coù : σ = ⎜⎜ ⎟⎟ suy ra σ 2 = ⎜⎜ ⎟⎟ ⎝ 3421⎠ ⎝ 2143 ⎠ Nhaän thaáy : σ laø moät chu trình nhöng σ 2 khoâng phaûi laø moät chu trình. Bài 1.17: Cho (G,*) la một nhóm và H là một nhóm con của G.Chứng minh rằng với x ∈ G Các khẳng định sau là tương đương: a) xH là một nhóm con của G; b) Hx là một nhóm con của G; c) x ∈ H. Giải : • (a => b): Với mọi xh1,xh2 ∈ xH , (xh1)-1* (xh2) ∈ xH , trong đó h1,h2 ∈ H Suy ra tồn tại h ∈ H sao cho (xh1)-1* (xh2) = xh hay : x-1*(xh1)-1* (xh2)*x =hx x-1h-1x-1xh2x = hx (hx)-1h2x = hx ∈ Hx • (b => c): Với mọi h1x,h2x ∈ Hx , (h1x)-1* (h2x) ∈ Hx , trong đó h1,h2 ∈ H 4
  5. Suy ra tồn tại t ∈ H sao cho (h1x)-1* (h2x) = tx hay x-1h1-1h2 = t h1-1h2t-1 = x ∈ H do H là một nhóm con của G • (c => a): Với mọi xh1,xh2 ∈ xH = H, (xh1)-1* (xh2)∈ H = xH (bài 1.3) trong đó h1,h2 ∈ H. Baøi 1.21: Giaûi: a) Ta coù : (AB)C = {xc | x ∈ AB , c ∈ C}={abc | a ∈ A,b ∈ B , c ∈ C} A(BC) ={ay | a ∈ A , y ∈ BC}={abc | a ∈ A,b ∈ B , c ∈ C} Do vaäy : (AB)C = A(BC) b) Ta coù: A −1 = { a −1 | a ∈ A } (A −1 ) −1 = { a −1 | a ∈ A −1 } ={ (b −1 ) −1 | b ∈ A} ={ b | b ∈ A }= A c) Ta coù : AB = { ab | a ∈ A , b ∈ B } (AB) −1 = { x −1 | x ∈ AB} ={(ab) −1 | a ∈ A , b ∈ B } ={ b −1 .a −1 | a ∈ A , b ∈ B }={ y.z | y ∈ B −1 , z ∈ A −1 }= B −1 . A −1 d) • AA = A ⇔ {ab | a ∈ A, b ∈ A} = A ⇔ ab ∈ A, ∀a ∈ A, b ∈ A A−1 = A ⇔ {a −1 | a ∈ A} = A ⇔ a −1 ∈ A, ∀a ∈ A Töø hai ñieàu treân suy ra (ii) töông ñöông (i) • A−1 A = {ab | a ∈ A−1 , b ∈ A} = {a −1b | a ∈ A, b ∈ A} = A ⇔ a −1b ∈ A, ∀a, b ∈ A Ñieàu naøy noùi leân raèng (iii) töông ñöông (i). e) • G/s AB laø nhoùm con cuûa G, ta cm AB=BA +Laáy x thuoäc AB ta chöùng minh x thuoäc BA, thaät vaäy vì x thuoäc AB maø AB laø nhoùm con cuûa G neân x −1 thuoäc AB, suy ra x −1 = ab vôùi a thuoäc A,b thuoäc B. Vaäy x= b −1a −1 vôùi b −1 ∈ B, a −1 ∈ A suy ra x thuoäc BA. +Laáy x thuoäc BA chöùng minh x thuoäc AB (chöùng minh töông töï) • G/s AB=BA. Chöùng minh AB laø nhoùm con cuûa G. +Laáy a thuoäc A, b thuoäc B suy ra ab thuoäc AB neân AB khaùc troáng +Laáy x,y thuoäc AB baát kì ta cm xy thuoäc AB x=ab (a thuoäc A, b thuoäc B) y=cd (c thuoäc A, d thuoäc B) xy=abcd=a(bc)d Vì bc thuoäc BA maø BA=AB neân toàn taïi b’ thuoäc A, c’ thuoäc b sao cho bc=b’c’ Vaäy xy=(ab’)(c’d) thuoäc AB (vì ab’ thuoäc A, c’d thuoäc B) +Laáy x thuoäc AB ta chöùng minh x −1 thuoäc AB. x=ab (a thuoäc A, b thuoäc B) do ñoù x −1 = b −1a −1 ∈ BA = AB 5
  6. • Chöùng minh AB =< A ∪ B > +Vôùi moïi a thuoäc A ta coù a=ae thuoäc AB neân A laø con cuûa B, töông töï B laø con cuûa AB neân A ∪ B ⊂ AB . Vaäy < A ∪ B >⊂ AB +Hôn nöõa laáy x thuoäc AB baát kì thì x=ab vôùi a thuoäc A, b thuoäc B. Noùi rieâng thì x=ab vôùi a,b thuoäc A ∪ B do ñoù x thuoäc < A ∪ B > . Vaäy AB ⊂< A ∪ B > Bài 1.25: Cho nhóm (G,.) hữu hạn và H,K là hai nhóm con của G. Chứng minh rằng: HK H ∩ K = H K (1) Giải: • Vì G hữu hạn nên ta có H , K , H ∩ K , HK hữu hạn. • Hơn nữa vì H,K là các nhóm con của (G,.) cho nên: H ∩ K ≤ H, H ∩ K ≤ K . Theo định lí Lagrange ta có H = H :H ∩K H ∩K K = K :H ∩K H ∩K Từ đó suy ra H ∩ K là ước của H và H ∩ K là ước của K do đó nếu gọi k là số phần tử của H ∩ K thì số phần từ của H là nk (n ∈ N * ) và của K là mk (m ∈ N * ) . Nói cách khác ta có H ∩ K =k, H = nk , K = mk (2) Cho nên để chứng minh (1) ta chỉ cần chứng minh: HK = nmk là xong. • Chứng minh HK = nmk : +Lưu ý: HK = {hk: h ∈ H,k ∈ K} . Xét quan hệ tương đương trong H như sau: a, b ∈ H , khi đó a ∼ b được đ/n là a,b ∈ H , a −1b ∈ H ∩ K . Số lớp tương đương trong H với định nghĩa trên là H / H ∩ K = H / H ∩ K = nk / k = n +Cho a, b ∈ H giả sử a ∼ b khi đó a −1b ∈ H ∩ K và a −1b = (a −1b) −1 = b −1a ∈ H ∩ K Vì K = mk nên có thể đặt K = {k1 , k2 ,..., kmk } . Đặt A = {ak1 , ak2 ,..., akmk } và B = {bk1 , bk2 ,..., bkmk } . Ta có: A=B (g/s aki ∈ A đặt k j = b −1aki ∈ K do đó aki = bk j ∈ B nên A ⊂ B , chứng minh tương tự cho B ⊂ A ). Như vậy cứ mỗi một lớp tương đương sẽ cho tương ứng với mk phần tử có dạng hk (h ∈ H , k ∈ K ) (3) + Cho a, b ∈ H , giả sử a và b không cùng một lớp tương đương .Giả sử tồn tại i,j sao cho aki = bk j khi đó a −1b = ki k j −1 ∈ K ∩ H (vì a −1b ∈ H , ki k j −1 ∈ K ) nhưng như vậy thì a và b đã cùng một lớp tương đương (mâu thuẫn điều g/s) như vậy ta có aki ≠ bk j với mọi i,j (4) +Từ các kết luận trong (3) và (4) suy ra số phần tử của HK bằng số lớp tương đương nhân với mk hay là HK = nmk (đpcm). 6
  7. Bài 1.29: Tìm hai phần tử a,b của nhóm G sao cho a,b đều có cấp hữu hạn nhưng ab lại có cấp vô hạn. Giải: Xét không gian G=GL(2,n). ⎛ 0 −1⎞ ⎛1 2⎞ a=⎜ ⎟,b = ⎜ ⎟ , ta có a = b = 1 ≠ 0 nên a, b ∈ G . ⎝1 1 ⎠ ⎝ −1 −1⎠ n ⎛ 1 1⎞ ⎛ 1 n ⎞ Ta có a = b = I 2 nên a,b có cấp hữu hạn, tuy nhiên ( ab ) n 6 4 =⎜ ⎟ =⎜ ⎟ ≠ I 2 với ⎝ 0 1⎠ ⎝ 0 1 ⎠ mọi n ∈ N * nên ab có cấp vô hạn. Bài 1.33: Chứng minh: a) Mọi nhóm cyclic đều giao hoán b) Mọi nhóm con của một nhóm cyclic cũng cyclic c) Ảnh đồng cấu của một nhóm cyclic cũng cyclic Giải: a) Ta có: a = {a m | m ∈ Z } , do đó lấy u , v ∈ a bất kì, thì u,v có dạng u = a k , v = a l (Với k , l ∈ Z ) Suy ra uv = a k a l = a k +l = a l + k = a l a k = vu . b) Tính chất căn bản c) Xét đồng cấu f : G → U ( ở đây G = a ) Ta sẽ chứng minh f (G ) = {[f(a)]m | m ∈ Z } • Thật vậy xét u ∈ f (G ) bất kì, khi đó f(u) có dạng f (u ) = f (a n ) với n ∈ Z . Vì f đồng cấu cho nên f (a n ) = [f (a)]n , từ đó suy ra f (u ) = [f (a)]n ∈ {[f(a)]m | m ∈ Z } Vậy f (G ) ⊂ {[f(a)]m | m ∈ Z } (1) • Ngược lại, giả sử v ∈ {[f(a)]m | m ∈ Z } tức v = [f(a)]n , n ∈ Z Do f đồng cấu nên f (a n ) = [f (a)]n suy ra v = f (a n ) ∈ f (G ) Vậy f (G ) ⊃ {[f(a)]m | m ∈ Z } (2) (1)+(2) cho ta kết luận. Bài 1.37 Cho (G,.) là một nhóm và H là một nhóm con của G . Chuẩn hoá tử của H trong G là tập con của G định bởi N G ( H ) = {x ∈ G|xH=Hx} . Chứng minh rằng: a) N G ( H ) là nhóm con của G b) H là nhóm con chuẩn tắc của NG ( H ) c) H là nhóm con chuẩn tắc của G khi và chỉ khi N G ( H ) =G d) N G ( H ) là nhóm con lớn nhất của G nhận H làm nhóm con chuẩn tắc. Giải: 7
  8. a) • Vì e ∈ N G ( H ) nên N G ( H ) ≠ φ và đương nhiên ta có N G ( H ) ⊂ G . Như vậy ta cần chứng minh hai điều sau: i)Nếu u, v ∈ N G ( H ) thì uv ∈ N G ( H ) ii)Nếu u ∈ N G ( H ) thì u −1 ∈ N G ( H ) • G/s u, v ∈ N G ( H ) tức là uH=Hu và vH=hv: Suy ra uvH=u(vH)=u(Hv)=(uH)v=Huv nên uv ∈ N G ( H ) do đó (i) được chứng minh. • G/s u ∈ N G ( H ) tức là uH=Hu. Ta có eH=He nên uu −1 H = Huu −1 = ( Hu )u −1 = uHu −1 suy ra u −1 H = Hu −1 (luật giản ước) hay u −1 ∈ N G ( H ) vậy (ii) được chứng minh. b) • e∈ H ⇒ H ≠ φ • x ∈ H ⇒ xH = H = Hx ⇒ x ∈ N G ( H ) do đó H ⊂ N G ( H ) • x, y ∈ H ⇒ xy ∈ H (vì H là nhóm con của G) • x ∈ H ⇒ xH −1 = xG −1 (vì H là nhóm con của G) (1) x ∈ H ⇒ x ∈ N G ( H ) (vì H ⊂ N G ( H ) ) do đó xNG ( H ) −1 = xG −1 (vì N G ( H ) ≤ G ) (2) (1)+(2) suy ra xH −1 = xNG ( H ) −1 Từ các điều trên suy ra H ≤ N G ( H ) • Chứng minh H N G ( H ) Lấy x ∈ N G ( H ) bất kì, cần chứng minh xhx −1 ∈ H , ∀h ∈ H Ta có xH = Hx ⇒ xh = h ' x (h, h ' ∈ H ) suy ra xhx −1 = h ' ∈ H (xong!) c) • If NG ( H ) = G , theo câu b ta có H N G ( H ) suy ra H G • If H G ta cần chứng minh N G ( H ) = G . Đương nhiên ta có N G ( H ) ⊂ G ta sẽ chứng minh G ⊂ H nữa là xong Lấy x ∈ G bất kì, vì H G cho nên xH=Hx suy ra x ∈ N G ( H ) (xong!) d) Giả sử U ≤ G và H U ta sẽ chứng minh U ⊂ N G ( H ) . Thật vậy, lấy u ∈ U bất kì, vì H U nên ta có uH=Hu suy ra u ∈ N G ( H ) (xong!) Bài 1.41: Cho G là một nhóm hữu hạn và H là một nhóm con của G có chỉ số [G : H ] =2.Chứng minh H là một nhóm con chuẩn tắc của G .Hãy tổng quát hoá kết quả trên. Giải: Ta chỉ phát biểu và giải bài toán tổng quát. 8
  9. Bài toán:Giả sử p là số nguyên tố bé nhất chia hết cấp của nhóm G và H là nhóm con chỉ số p trong G .Chứng minh H G . Chứng minh: Tập hợp N H = { x ∈ G | xH = Hx} là chuẩn hoá tử của H trong G . Ta có H N H ≤ G và H G ⇔ N H = G (bài 1.37). Vậy để chứng minh H G ta sẽ chứng minh N H = G . Mà G hữu hạn và G = N H [G : N H ] (định lý Lagrange) nên điều cần chứng minh tương đương với G = N H hay [G : N H ] =1. Cũng theo định lý Lagrange ta có: G = H [G : H ] , N H = H [ N H : H ] (1). Suy ra: [G : H ] = [G : N H ][ N H : H ] (2). Do đó [G : N H ] là ước của [G : H ] .Mà [G : H ] = p là số nguyên tố nên [G : N H ] =1 hoặc [G : N H ] = p . • Nếu [G : N H ] =1 thì ta có điều phải chứng minh. • Nếu [G : N H ] = p thì từ (2) ta được [ N H : H ] =1.Lại từ (1) cho ta NH = H . Mà H ⊂ N H ⊂ G hữu hạn nên suy ra N H = H . Đặt P = { xHx −1 | x ∈ G} . Xét ánh xạ g : G / N H → P xN H xHx −1 ∀x, y ∈ G ta có: g ( xN H ) = g ( yN H ) ⇔ xHx −1 = yHy −1 ⇔ ( y −1 x) H = H ( y −1 x) ⇔ y −1 x ∈ N H ⇔ xN H = yN H . Chiều (⇐) chứng tỏ g là ánh xạ,chiều (⇒) cho thấy g là đơn ánh. Hơn nữa, g là toàn ánh vì với mọi V ∈ H thì V = xHx −1 ( x ∈ G ) ,chọn U = xN H ∈ G / N H ta được g (U ) = V . Vậy g là song ánh.Suy ra: P = G : N H = p . Kí hiệu S p là tập hợp các song ánh từ P vào P .Thế thì S p là nhóm với phép hợp nối ánh xạ,đồng thời S p = p ! . Với mỗi a ∈ G xét ánh xạ: fa : P → P xHx −1 axHx −1a −1 ∀x, y ∈ G ta có: f a ( xHx −1 ) = f a ( yHy −1 ) ⇔ axHx −1a −1 = ayHy −1a −1 ⇔ xHx −1 = yHy −1 . Suy ra f a là ánh xạ và là đơn ánh. Hơn nữa,nếu V = yHy −1 ∈ P thì V = a(a −1 y ) H (a −1 y ) −1 a −1 .Chọn x = a −1 y;U = xHx −1 ta được f a (U ) = V nên f a là toàn ánh. Vậy f a là song ánh.Do đó f a ∈ S p , ∀ a ∈ G . Xét ánh xạ: f : G → S p 9
  10. a fa Ta có f (ab) = f ab ; f (a) f (b) = f a fb . ∀xHx −1 ∈ P thì f a f b ( xHx −1 ) = f a (bxHx −1b −1 ) = abxHx −1b −1a −1 = f ab ( xHx −1 ) nên f ab = f a f b hay f (ab) = f (a ) f (b) .Do đó f là một đồng cấu nhóm. Nếu a ∈ kerf thì f a = Id P .Cho nên f a ( z ) = z, ∀z ∈ P .Vì H ∈ P do đó f a ( H ) = H hay aHa −1 = H ⇔ aH = Ha ⇔ a ∈ N H = H .Do vậy kerf ⊂ H .Mà kerf là nhóm nên kerf ≤ H . Ta chứng minh [ H : ker f ] =1. Giả sử [ H : ker f ] >1,thế thì [ H : ker f ] có một ước số nguyên tố q . Ta có [G : ker f ] = G / ker f = ( H [G : H ]) /( H / [ H : ker f ]) = [G : H ][ H : ker f ] pq . Mà G / ker f ≅ Im f ≤ SP (định lý 8.9 và 8.7). Suy ra p ! pq ⇒ ( p − 1)! q .Do q là số nguyên tố nên phải có q < p . Nhưng q | [ H : ker f ] thì cũng có q | G ,mâu thuẫn với giả thiết p là ước số nguyên tố nhỏ nhất chia hết cấp G . Vậy [ H : ker f ] =1.Suy ra H = ker f .Lại có kerf ⊂ H hữu hạn nên H = ker f . Mà ker f G (định lý 8.7) do đó H G .Theo bài 1.37 thì N H = G ⇒ [G : N H ] =1(mâu thuẫn). Vậy bài toán giải quyết xong. Bài 1.45: Chứng minh rằng: a) Nhóm hoán vị Sn được sinh bởi các chuyển vị b) Nhóm thay phiên An là nhóm con chuẩn tắc của Sn và được sinh bởi các 3-chu trình c) Nếu H là một nhóm con chuẩn tắc của An và H có chứa ít nhất một 3-chu trình thì H= An Giải: a) Gọi A là tập chứa tất cả các chuyển vị, đương nhiên ta có A ⊂ Sn , vì Sn là một nhóm và chứa A cho nên A ⊂ S n (1) Nhưng lưu ý với σ ∈ Sn bất kì thì σ luôn được phân tích dưới dạng tích của các chuyển vị, do đó σ ∈ A , vì vậy S n ⊂ A (2) Từ (1) và (2) suy ra nhóm hoán vị Sn được sinh từ các chuyển vị. b) • Nhắc lại nhóm thay phiên An là tập hợp tất cả các hoán vị chẵn của nhóm hoán vị Sn . Ta đã biết là An ≤ Sn . Bây giờ ta sẽ chứng minh A Sn . Muốn vậy lấy σ ∈ S n bất kì ta 10
  11. cần chứng minh σ rσ −1 ∈ An , ∀r ∈ An . Thật vậy: sgn(σ rσ −1 ) = sgn(σ ).sgn(r ).sgn(σ −1 ) = sgn(σ ).sgn(σ ) −1 sgn( r ) = sng ( r ) = 1 Do đó σ rσ −1 ∈ An (đpcm) • Bây giờ ta sẽ chứng minh An được sinh bởi các 3-chu trình. Gọi B là tập hợp các 3-chu trình. Ta cần chứng minh B = An . +Giả sử u ∈ B bất kì, khi đó u = σ 1 'σ 2 '...σ q ' trong đó các σ i ' là các 3-chu trình. Lưu ý là sgn(σ i ') = 1, ∀i do đó sgn(u ) = 1 suy ra u ∈ An . Vậy B ⊂ An (1) +Tiếp theo ta sẽ chứng minh An ⊂ B , thật vậy: Gọi σ ∈ An bất kì, ta có thể phân tích σ thành tích của các chu trình σ i trong Sn : σ = σ 1σ 2 ...σ m . Hơn nữa mỗi σ i lại có thể phân tích dưới dạng tích của các chuyển vị trong Sn : σ i = (r1r2 ...rp ) = (r1rp )(r1rp −1 )...(r1r2 ) . Chính vì vậy ta có thể phân tích σ thành tích của các chuyển vị, trong đó hai chuyển vị đứng cạnh nhau có dạng: (ri rj )(ri rk ) (trong đó ri , rj , rk khác nhau đôi một), (2) hoặc là: (ri rj )(rk rl ) (trong đó ri , rj , rk , rl khác nhau đôi một). (3) Hơn nữa sgn(σ ) = 1 nên số chuyển vị phân tích như trên là một số chẳn, do đó ta có thể phân tích σ thành tích của các cặp chuyển vị mà mỗi cặp có dạng (2) hoặc dạng (3). Lưu ý là (ri rj )(rk rl ) = (ri rk rl ) và (ri rj )(rk rl ) = (rj rk rl )(ri rl rj ) . Điều này cho ta kết luận có thể phân tích σ thành tích của các 3-chu trình. Vậy An ⊂ B (4) Từ (1) và (4) cho ta kết luận B = An . c) • Giả sử H An và H chứa một 3-chu trình là (abc) . Cần chứng minh H = An • Nhận xét: Nếu ( xyz ) ∈ H thì ( xyz ), ( yzx), ( zxy ), ( xzy ), ( yxz ), ( zyx ) ∈ H Thật vậy vì ( xzy ) = ( xyz )( xyz ) thuộc H. Mà ta có: ( xyz ) = ( yzx) = ( zxy ) và ( xzy ) = ( yxz ) = ( zyx) . Nên nhận xét được chứng minh. • Trở lại bài toán, lấy p ∈ {1,2,...,n} bất kì thoả p ∉ {a,b,c} . Ta có: ( pcb) = ( pac)(abc)( pac) −1 ∈ H . Theo “nhận xét” suy ra: (bpc) ∈ H . Lập luận tương tự như trên, với mọi q ∉ {b,c,p} ta có: (qpc) ∈ H . Lại theo “nhận xét” suy ra (cqp ) ∈ H , và lại lập luận tương tự ta có: (rqp) ∈ H , ∀r ∉ {p,q,c} Nói riêng ta có: (rqp) ∈ H , ∀p, q, r ∉ {a,b,c} trong đó p,q,r đôi một khác nhau. Kết hợp điều này với việc (abc) ∈ H và “nhận xét” suy ra mọi 3-chu trình đều thuộc H, mà theo câu b ta đã biết mọi σ ∈ An luôn phân tích được dưới dạng tích của các 3-chu trình do đó An ⊂ H , theo giả thiết ta lại có H An . Vậy bắt buộc H = An (đpcm) Bài 1.49: 11
  12. Xét đồng cấu nhóm cộng f : Q → Z a)Chứng minh rằng với n ∈ N * thì f (1) = nf (1/ n) b)Suy ra: f(1)=0 và f phải là đồng cấu tầm thường. Giải: a)Với n ∈ N * , ta có: ⎛ ⎞ ⎛n⎞ ⎜1 1 1⎟ ⎛1⎞ ⎛1⎞ ⎛1⎞ ⎛1⎞ f (1) = f ⎜ ⎟ = f ⎜ + + ... + ⎟ = f ⎜ ⎟ + f ⎜ ⎟ + ... + f ⎜ ⎟ = nf ⎜ ⎟ ⎝n⎠ ⎜n n n⎟ ⎝n⎠ ⎝n⎠ ⎝n⎠ ⎝n⎠ ⎝ n ⎠ n b) ⎛1⎞ ⎛ 1 ⎞ f (1) • Theo câu (a) ta có f (1) = nf ⎜ ⎟ , ∀n ∈ N * , suy ra f ⎜ ⎟ = , ∀n ∈ N * (1) n ⎝ ⎠ n ⎝ ⎠ n Giả sử f (1) ≠ 0 khi đó thay n = 2 f (1) ∈ N vào (1) ta có: * ⎛ 1 ⎞ f (1) f ⎜⎜ ⎟⎟ = ∉ Z (mâu thuẫn f : Q → Z ) ⎝ 2 f (1) ⎠ 2 f (1) ⎛1⎞ ⎛1⎞ Vậy f (1) = 0 , do đó từ f (1) = nf ⎜ ⎟ , ∀n ∈ N * suy ra f ⎜ ⎟ = 0, ∀n ∈ N * . ⎝n⎠ ⎝n⎠ • Lấy u ∈ Q bất kì ta sẽ chứng minh f(u)=0. +Ta có f (0) = f (0 + 0) = f (0) + f (0) ⇒ f (0) = 0 , nên u=0 thoả. Hơn nữa f (u ) + f (−u ) = f (u − u ) = f (0) = 0 ⇒ f (u ) = f (−u ), ∀u ∈ Q , do đó ta chỉ cần chứng minh f (u ) = 0, ∀u ∈ {x ∈ Q|x>0} là xong. m +Xét u ∈ {x ∈ Q|x>0} bất kì tức là u có dạng u = với m, n ∈ N * , ta cần chứng minh n f(u)=0. Thật vậy: ⎛ ⎞ ⎛m⎞ ⎜1 1 1⎟ ⎛1⎞ ⎛1⎞ ⎛1⎞ ⎛1⎞ ⎛1⎞ f ⎜ ⎟ = f ⎜ + + ... + ⎟ = f ⎜ ⎟ + f ⎜ ⎟ + ... + f ⎜ ⎟ = mf ⎜ ⎟ = 0 (vì f ⎜ ⎟ = 0 ) ⎝n⎠ ⎜n n n⎟ ⎝n⎠ ⎝n⎠ ⎝n⎠ ⎝n⎠ ⎝n⎠ ⎝ m ⎠ m Vậy f(u)=0 , ∀u ∈ Q tức f là đồng cấu tầm thường ! Bài 1.53: Chứng minh rằng tồn tại duy nhất (sai khác một đẳng cấu) một nhóm hữu hạn cấp 8 sinh bởi hai phần tử a,b thoả hệ thức: a 4 = e, ba = a −1b , a 2 = b 2 Nhóm này được gọi là nhóm Quaternion. Giải: Trước hết ta có các kết quả sau: i) Nếu a,b là hai phần tử của nhóm hữu hạn G sao cho là nhóm thì =. 12
  13. Thật vậy,vì a, b ∈< a >< b > nên < a, b >⊂< a >< b > .Mặt khác ,với mọi x ∈< a >< b > thì x = x1 x2 với x1 = a m ∈< a >, x2 = b n ∈< b > nên x = a mb n ∈< a, b > .Do đó < a >< b >⊂< a, b > .Vậy < a >< b >=< a, b > . ii) Giả sử G là nhóm hữu hạn sinh bởi hai phần tử a, b thoả hệ thức a 4 = e, ba = a −1b, a 2 = b 2 ( ⇒ b 4 = e ). Ta có : < a >= {e, a, a 2 , a 3 } , < b >= {e, b, b 2 , b3 } , < a >< b >= {e, a, a 2 , a 3 , b, ab, a 2b, a 3b} = {e, a, a 2 , a −1 , b, ab, b −1 , ba} Kiểm tra được ∀x, y ∈< a >< b > thì x −1 , xy ∈< a >< b > do đó < a >< b > là nhóm. Theo i) ta được G =< a, b >=< a >< b > . Suy ra G = < a >< b > ≤ 8 . Trở lại bài toán • C/m tồn tại duy nhất (sai khác một đẳng cấu) Giả sử G là nhóm hữu hạn cấp 8 sinh bởi hai phần tử a, b thoả a 4 = e, ba = a −1b, a 2 = b 2 P là nhóm hữu hạn cấp 8 sinh bởi hai phần tử c, d thoả c 4 = e, dc = c −1d , c 2 = d 2 Từ ii) suy ra G = {e, a, a 2 , a −1 , b, ab, b −1 , ba} , P = {e, c, c 2 , c −1 , d , cd , d −1 , dc} Xét ánh xạ f : G → P x f ( x) sao cho f ( x) = e, c, c 2 , c −1 , d , cd , d −1 , dc khi tương ứng x = e, a, a 2 , a −1 , b, ab, b −1 , ba Kiểm tra ta thấy f là một đẳng cấu. • C/m sự tồn tại Xét nhóm con Q của nhóm GL(2,C) sinh bởi hai ma trận ⎛ 0 1⎞ ⎛0 i ⎞ a=⎜ ⎟,b = ⎜ ⎟ ⎝ −1 0 ⎠ ⎝ i 0⎠ ⎛ 0 1⎞ −1 ⎛ −i 0 ⎞ 2 ⎛ −1 0 ⎞ Ta có: a 4 = ⎜ ⎟ , ba = a b = ⎜ ⎟,a = b = ⎜ 2 ⎟ ⎝ −1 0 ⎠ ⎝ i 0⎠ ⎝ 0 −1 ⎠ ⎛ 0 −1⎞ −1 ⎛ 0 −i ⎞ ⎛i 0 ⎞ Nên theo ii) suy ra Q ≤ 8 .Ta lại có a −1 = ⎜ ⎟,b = ⎜ ⎟ , ab = ⎜ ⎟ ⎝1 0 ⎠ ⎝ −i 0 ⎠ ⎝ 0 −i ⎠ Do đó tập hợp {e, a, a 2 , a −1 , b, ab, b −1 , ba} có 8 phần tử và chứa trong Q nên Q ≥ 8 , Vậy Q = 8 và Q = {e, a, a 2 , a −1 , b, ab, b −1 , ba} . Bài 1.57: a) Cho G là một nhóm hữu hạn có cấp |G| ≤ 5.Chứng minh G giao hoán. b) Cho G là một nhóm hữu hạn có cấp 6.Chứng minh rằng nếu G giao hoán thì G là nhóm cyclic đẳng cấu với 6 ; nếu G không giao hoán thì G đẳng cấu với nhóm nhị diện D3 . Giải: 13
  14. a)Cho G là một nhóm hữu hạn có cấp ≤ 5 . -Nếu G = 1 thì G = {e} nên G giao hoán. -Nếu G ∈ {2,3,5} thì G có cấp nguyên tố nên G cyclic,do đó G giao hoán. -Xét khi G = 4 +Nếu G có một phần tử a có cấp 4 thì G =< a > cyclic nên G giao hoán. +Nếu G không có phần tử cấp 4 thì ba phần tử khác e của G đều có cấp 2,suy ra G giao hoán (Bài 1.6). Vậy nếu G là nhóm hữu hạn có cấp ≤ 5 thì G giao hoán. b)Vì G cấp 6 nên G chứa một phần tử a có cấp 2,và một phần tử b có cấp 3. • Nếu G giao hoán thì ab = ba .Suy ra c = ab có cấp 6 (bài 1.30 c) ). Do đó G =< c > là nhóm cyclic và G ≅ 6 (hệ quả 8.12; Bài 1.52). • Nếu G không giao hoán thì ab ≠ ba (vì nếu ab = ba thì c = ab có cấp 6 và G =< c > cyclic dẫn đến G giao hoán). Suy ra: < a > ∩ < b >= {e} ,và ab, ba ∉< a >, < b > . Do đó G = {e, a, b, b 2 , ab, ba} , G = 6 . Xét phần tử b −1a ∈ G ,ta thấy b −1a ∉ {e, a, b, b 2 , ba} nên b −1a = ab . Như vậy G là nhóm hữu hạn cấp 6=2.3 sinh bởi hai phần tử a, b thoả a 2 = e, b3 = e, ab = b −1a nên theo bài 1.55 thì G ≅ D3 . Bài 1.61: Chứng minh rằng nhóm G ' là ảnh đồng cấu của một nhóm cyclic hữu hạn G khi và chỉ khi G ' là nhóm cyclic hữu hạn có cấp chia hết cấp G . Giải: • Cho G ' là ảnh đồng cấu của một nhóm cyclic hữu hạn G .C/m G ' là nhóm cyclic hữu hạn có cấp chia hết cấp G . Theo giả thiết G là nhóm cyclic hữu hạn,giả sử G =< a > , G = m và có một toàn cấu f : G → G' . Đặt b = f (a) ,ta có < b >⊂ G ' . Với mọi y ∈ G ' tồn tại x ∈ G sao cho y = f ( x) . Vì x ∈ G =< a > nên tồn tại n ∈ sao cho x = a n . Suy ra y = f ( x) = f (a n ) = f n (a) = b n .Do đó y ∈< b >⇒ G ' =< b > . Vậy G ' là nhóm cyclic. Mặt khác do b m = f m (a) = f (a m ) = f (e) = e' nên G ' có cấp hữu hạn.Đặt k =< b >= G ' . Do b m = e' nên suy ra k \ m .Hay G ' có cấp chia hết cấp G . • Cho G, G ' là các nhóm cyclic hữu hạn,| G ' | \ | G |.C/m G ' là ảnh của G qua một đồng cấu nào đó. Giả sử G =< a > và G = m G ' =< b > và G ' = n ,với n \ m . 14
  15. Xét ánh xạ f : G → G ' at bt ' ' ' f xác định vì nếu a t = a t thì a t −t = e nên m \ (t − t ' ) .Suy ra n \ (t − t ' ) ,do đó bt −t = e' ' ' hay bt = bt hay f (a t ) = f (a t ) . Với mọi z ∈ G ' , z = bt (t ∈ ) ,chọn x = a t thì z = f ( x) nên f là toàn ánh. Với mọi x, y ∈ G, x = a r , y = a s (r , s ∈ ) . Ta có f ( x) f ( y ) = b r b s = b r + s = f (a r + s ) = f ( xy ) .Suy ra f là đồng cấu. Vậy G ' là ảnh đồng cấu của G qua f . Bài 1.65: Cho f là một đẳng cấu từ nhóm (G,.) đến nhóm (G ' ,.) . a) Chứng minh rằng tương ứng H f ( H ) là một song ánh giữa tập hợp các nhóm con ' của G và G . b) Chứng minh rằng tương ứng H f ( H ) là một song ánh giữa tập hợp các nhóm con ' chuẩn tắc của G và G . c) Giả sử H là một nhóm con chuẩn tắc của G .Chứng minh rằng tương ứng xH f ( x) H là một đẳng cấu từ nhóm thương G / H đến nhóm thương G ' / f ( H ) . Giải: a) Kí hiệu P( X ) là tập hợp tất cả các nhóm con của nhóm X . Ta cần chứng minh ánh xạ: g : P(G ) P (G ' ) H f (H ) là một song ánh. • Ta có, nếu H ≤ G thì f ( H ) ≤ G ' (Định lí 8.7). Hơn nữa nếu H = K ≤ G .Cho y ∈ f ( H ) ,tồn tại x ∈ H = K sao cho y = f ( x) ∈ f ( K ) ,do đó f ( H ) ⊂ f ( K ) .Tương tự f ( K ) ⊂ f ( H ) .Cho nên f ( H ) = f ( K ) . Vậy ánh xạ g là hoàn toàn xác định. • Nếu H , K ≤ G và f ( H ) = f ( K ) .Cho x ∈ H ,dẫn đến f ( x) ∈ f ( H ) = f ( K ) ,suy ra x ∈ K ,nên H ⊂ K .Tương tự K ⊂ H .Do đó H = K . Vậy ánh xạ g là đơn ánh. • Với mọi U ∈ P(G ' ) ,ta có U ≤ G ' ,nên f −1 (U ) ≤ G (Định lí 8.7). Đặt V = f −1 (U ) thì V ∈ P(G ) và g (V ) = f (V ) = U .Vậy g ;là toàn ánh. Từ các điều trên ta có g là một song ánh. b) Kí hiệu S ( X ) là tập hợp tất cả các nhóm con chuẩn tắc của nhóm X . Ta cần chứng minh ánh xạ h : S (G ) S (G ' ) H f (H ) là một song ánh. • Ta có nếu H G ,thì H ≤ G nên f ( H ) ≤ G ' (Định lí 8.7).Mặc khác, với mọi v ∈ G ' , y ∈ f ( H ) ;tồn tại u ∈ G, x ∈ H sao cho v = f (u ), y = f ( x) .Vì H G nên 15
  16. u −1 xu ∈ H , dẫn đến f −1 (u ) f ( x) f (u ) = f (u −1 xu ) ∈ f ( H ) ,hay v −1 yv ∈ G ' .Do đó f (H ) G' . Hơn nữa nếu H = K G thì tương tự câu a) ta cũng có f ( H ) = f ( K ) . Vậy h xác định . • Tương tự câu a) ta cũng có nếu H , K G và f ( H ) = f ( K ) thì H = K .Vậy h là đơn ánh. • Với mọi U ∈ S (G ' ) ,ta có U G ' nên f −1 (U ) G (Định lí 8.7). Đặt H = f −1 (U ) thì H ∈ S (G ) và h( H ) = f ( H ) = U .Vậy h là toàn ánh. Từ các điều trên ta cũng được h là song ánh. c) Theo câu b) H G thì f ( H ) G ' . Ta phải chứng minh ánh xạ g : G / H G' / f (H ) xH f ( x) H là một đẳng cấu. • Với mọi xH , yH ∈ G / H ,ta có : g ( xH ) = g ( yH ) ⇔ f ( x) f ( H ) = f ( y ) f ( H ) ⇔ f −1 ( x) f ( y ) ∈ f ( H ) ⇔ f ( x −1 y ) ∈ f ( H ) ⇔ x −1 y ∈ H (do f là đẳng cấu) ⇔ xH = yH . Chiều ( ⇐ ) chứng tỏ g là một ánh xạ,chiều (⇒) chứng tỏ g đơn ánh. Mặt khác nếu cho zf ( H ) ∈ G ' .Đặt x = f −1 ( z ) .Ta có g ( xH ) = f ( x) f ( H ) = zf ( H ) .Nên g là toàn ánh. Vậy g là một song ánh. • Kiểm tra g là một đồng cấu. Với mọi xH , yH ∈ G / H .Ta có: g ( xH ) g ( yH ) = ( f ( x) f ( H ))( f ( y ) f ( H )) = ( f ( x) f ( y )) f ( H ) = f ( xy ) f ( H ) = g (( xy ) H ) . Vậy g là một đồng cấu. Tóm lại ta có g là một đẳng cấu từ G / H đến G ' / f ( H ) . Bài 1.69: a) Cho nhóm (G,.) và G1 , G2 ,..., Gn là các nhóm con chuẩn tắc của G thoả các điều kiện sau: G = G1G2 ...Gn và Gi ∩ (G1...Gi −1Gi +1...Gn ) = {e} , ∀1 ≤ i ≤ n . Chứng minh rằng: i ) Mọi phần tử x ∈ G được viết duy nhất dưới dạng x = x1 x2 ...xn trong đó xi ∈ Gi , ∀1 ≤ i ≤ n. ii ) G đẳng cấu với nhóm tích trực tiếp G1 × G2 × ... × Gn .Ta nói G là một nội tích trực tiếp của các nhóm con Gi ,1 ≤ i ≤ n . b) Chứng minh rằng nếu nhóm (G ,.) đẳng cấu với tích trực tiếp của những nhóm con H i ,1 ≤ i ≤ n thì trong G sẽ tồn tại các nhóm con Gi đẳng cấu với H i sao cho G là nội tích trực tiếp của các nhóm con Gi ,1 ≤ i ≤ n . Giải: 16
  17. Trước hết ta có:nếu H , K là các nhóm con chuẩn tắc trong G thì HK cũng là một nhóm con chuẩn tắc trong G (Bài 1.38). Bằng quy nạp ta suy ra nếu H1 , H 2 ,..., H m (m ∈ * ) là các nhóm con chuẩn tắc trong G thì H1 H 2 ...H m cũng là nhóm con chuẩn tắc trong G . a) i ) Cho x ∈ G ,Vì G = G1G2 ...Gn nên tồn tại ( x1 , x2 ,..., xn ) ∈ G1 × G2 × ... × Gn sao cho x = x1 x2 ...xn . Giả sử có ( y1 , y2 ,..., yn ) ∈ G1 × G2 × ... × Gn thoả x = y1 y2 ... yn .Ta chứng minh xi = yi , ∀1 ≤ i ≤ n . Thật vậy, từ x1 x2 ...xn = y1 y2 ... yn suy ra với mọi 1 ≤ i ≤ n ta có: { } yi −1 xi = yi −1 ⎡⎣( x1...xi −1 ) −1 ( y1... yi −1 ) ⎤⎦ yi ⎡⎣ ( yi +1... yn )( xi +1...xn ) −1 ⎤⎦ ∈ G1...Gi −1Gi +1...Gn .(Do G1...Gi −1 G và Gi +1...Gn G theo nhận xét trên). Mà yi −1 xi ∈ Gi và Gi ∩ (G1...Gi −1Gi +1...Gn ) = {e} , ∀1 ≤ i ≤ n nên suy ra yi −1 xi = e hay xi = yi , ∀1 ≤ i ≤ n . Vậy x ∈ G được viết duy nhất dưới dạng x = x1 x2 ...xn trong đó xi ∈ Gi , ∀1 ≤ i ≤ n. ii ) Xét ánh xạ f : G → G1 × G2 × ... × Gn x ( x1 , x2 ,..., xn ) với x1 , x2 ,..., xn thoả x = x1 x2 ...xn . Theo i ) với mỗi x ∈ G thì tồn tại duy nhất ( x1 , x2 ,..., xn )∈ G1 × G2 × ... × Gn sao cho x = x1 x2 ...xn ,nên ánh xạ trên xác định và là đơn ánh. Hơn nữa với mỗi ( x1 , x2 ,..., xn )∈ G1 × G2 × ... × Gn ,đặt x = x1 x2 ...xn thì f ( x) = ( x1 , x2 ,..., xn ) ,nên f là toàn ánh. Ta chứng minh f là một đồng cấu. Cho x, y ∈ G ;ta cần chứng minh f ( xy ) = f ( x) f ( y ) . Giả sử x = x1 x2 ...xn , y = y1 y2 ... yn ,trong đó xi , yi ∈ Gi ,1 ≤ i ≤ n . Ta được xy =( x1 x2 ...xn )( y1 y2 ... yn ).Mà xy ∈ G nên tồn tại z = z1 z2 ...zn ( zi ∈ Gi ) sao cho xy = z .Hay ( x1 x2 ...xn )( y1 y2 ... yn )= z1 z2 ...zn (1). Với mọi 1 ≤ i ≤ n ,ta có: { }{ zi −1 xi yi = ⎡⎣ zi −1 ( x1...xi −1 ) −1 ( z1...zi −1 ) zi ⎤⎦ ⎡⎣( zi +1...zn )( yi +1... yn ) −1 ⎤⎦ yi −1 ⎡⎣ ( y1... yi −1 ) −1 ( xi +1...xn ) −1 ⎤⎦ yi } ∈ G1...Gi −1Gi +1...Gn (Do G1...Gi −1 G và Gi +1...Gn G và G1...Gi −1Gi +1...Gn G theo nhận xét ban đầu). Mà zi xi yi ∈ G và Gi ∩ (G1...Gi −1Gi +1...Gn ) = {e} , ∀1 ≤ i ≤ n nên suy ra zi −1 xi yi = e .hay −1 zi = xi yi , ∀1 ≤ i ≤ n . Do đó ( x1 x2 ...xn )( y1 y2 ... yn )= ( x1 y1 )( x2 y2 )...( xn yn ) . Suy ra f ( xy ) = f ( x) f ( y ) = ( x1 y1 , x2 y2 ,..., xn yn ) . 17
  18. Như vậy f là một đẳng cấu nên ta được : G đẳng cấu với nhóm tích trực tiếp G1 × G2 × ... × Gn . b) Giả sử có những nhóm con H i ,1 ≤ i ≤ n và một đẳng cấu g : G → H1 × H 2 × ... × H n x (h1 , h2 ,..., hn ) Ta chứng minh có các nhóm con chuẩn tắc Gi , 1 ≤ i ≤ n sao cho G = G1G2 ...Gn , Gi ∩ (G1...Gi −1Gi +1...Gn ) = {e} , ∀1 ≤ i ≤ n và Gi đẳng cấu với H i . Khi đó G là tích nội trực tiếp của các nhóm con Gi , 1 ≤ i ≤ n theo kết quả ở ii ) . Với mọi 1 ≤ i ≤ n đặt K i = {(e1 ,..., ei −1 , a, ei +1 ,..., en ) | a ∈ H i } .trong đó ei = e , 1 ≤ i ≤ n . Ta có K i ≠ ∅ vì e' = (e1 , e2 ,..., en ) ∈ K i .( e' là ptdv của H1 × H 2 × ... × H n ). ∀u, v ∈ K i , u = (e1 ,..., ei −1 , a, ei +1 ,..., en ) , v = (e1 ,..., ei −1 , b, ei +1 ,..., en ) với a, b ∈ H i . Thì u −1v = (e1 ,..., ei −1 , a −1b, ei +1 ,..., en ) ∈ K i (do a −1b ∈ H i ).Nên K i ≤ H i . Hơn nữa với mọi z= ( z1 , z2 ,.., zn ) ∈ H1 × H 2 × ... × H n ta có : z −1uz = (e1 ,..., ei −1 , zi −1azi , ei +1 ,..., en ) ∈ K i (do zi −1azi ∈ H i ) nên K i Hi .1 ≤ i ≤ n . Đặt Gi = g −1 ( K i ) .Theo định lí 8.7 suy ra Gi G , 1 ≤ i ≤ n . Hơn nữa g|Gi : Gi → H1 × H 2 × ... × H n là đơn cấu và g|Gi (Gi ) = K i nên Gi ≅ K i . Đồng thời phép nhúng li : H i → H1 × H 2 × ... × H n a (e1 ,..., ei −1 , a, ei +1 ,..., en ) là đơn cấu và li ( H i ) = K i nên H i ≅ K i . Suy ra Gi ≅ H i , 1 ≤ i ≤ n . Cho x ∈ G ,tồn tại (h1 , h2 ,..., hn ) ∈ H1 × H 2 × ... × H n sao cho g ( x) = (h1 , h2 ,..., hn ) = (h1 , e2 ,..., en )(e1, h2 , e3 ,..., en )...(e1 ,..., en −1 , hn ) ∈ K1 × K 2 × ... × K n . Với mọi 1 ≤ i ≤ n tồn tại xi ∈ Gi sao cho g ( xi ) = (e1 ,..., ei −1 , hi , ei +1 ,..., en ) . Suy ra g ( x) = g ( x1 ) g ( x2 )...g ( xn ) = g ( x1 x2 ...xn ) . Do g đẳng cấu nên x = x1 x2 ...xn ∈ G1G2 ...Gn .Do đó G ⊂ G1G2 ...Gn . Mà G1G2 ...Gn ⊂ G . Vậy G = G1G2 ...Gn . 18
  19. 19
  20. 20
ADSENSE

CÓ THỂ BẠN MUỐN DOWNLOAD

 

Đồng bộ tài khoản
2=>2